LSAT and Law School Admissions Forum

Get expert LSAT preparation and law school admissions advice from PowerScore Test Preparation.

 Administrator
PowerScore Staff
  • PowerScore Staff
  • Posts: 9026
  • Joined: Feb 02, 2011
|
#98378
Complete Question Explanation

Justify the Conclusion. The correct answer choice is (B).

Answer choice (A):

Answer choice (B): This is the correct answer choice.

Answer choice (C):

Answer choice (D):

Answer choice (E):

This explanation is still in progress. Please post any questions below!
User avatar
 katnyc
  • Posts: 35
  • Joined: Dec 22, 2020
|
#100038
Hi there, can someone please go over this question in detail. I am understanding that this is a justify question but I cant place it for the right answer. I was between C D and E
User avatar
 Jeff Wren
PowerScore Staff
  • PowerScore Staff
  • Posts: 947
  • Joined: Oct 19, 2022
|
#100622
Hi kat,

This is a tricky question due to the use of the term "few children" and how to represent this idea when diagramming.

The argument can be boiled down to the following.

Premise: "Few children today spend their free time reading stories"

Conclusion: "Few children today will develop a lifelong interest in literature."

(The details about television are extraneous and irrelevant information.)

When the LSAT uses the term "few" here, what they are really getting at is the idea of "not many" or "most do not."

Here it can be helpful to rephrase this argument as:

Most children today do not spend their free time reading stories. Therefore, most children today will not develop a lifelong interest in literature.

With that wording, what we need to justify this argument is an answer that states:

If children do not spend their free time reading stories, then they will not develop a lifelong interest in literature.

Of course this prephrase could also appear in the form of the contrapositive, which would be:

If children will develop a lifelong interest in literature, then they must spend their free time reading stories.

Answer B basically matches this contrapositive in meaning (even though it uses "people" rather than "children," it still works.)

Since this is a justify question, the correct answer must get us 100% from the premise to the conclusion. Answer B is the only answer that does this. Justify questions often involve conditional reasoning, and this question requires a conditional answer that takes us from the premise to the conclusion (or the contrapositive of that conditional, which is what we get in Answer B).

Answer C is conditional, but it links lack of TVs to spending free time reading stories. This is not the gap in the argument that we need to fill and does not get us to our conclusion.

Answers D and E are not conditional statements and likewise do not close the gap in the argument. Basically, any answer mentioning TV is wrong here.
User avatar
 cemck6
  • Posts: 2
  • Joined: Sep 19, 2023
|
#103253
Can you explain why A is incorrect? I understood it as saying that if you spend free time reading stories that you will develop a lifelong interest in literature. I was between A and B.

Thank you!
User avatar
 Jonathan Evans
PowerScore Staff
  • PowerScore Staff
  • Posts: 727
  • Joined: Jun 09, 2016
|
#103344
Hi, cemck6,

Good question!

You have correctly identified the conceptual gap between reading stories and a lifelong interest in literature. Good job!

The difference between answer choices (A) and (B) is that answer choice (A) gives a lower limit and answer choice (B) gives an upper limit.

What I mean by that is that the conclusion is interested in the upper limit. We want to be sure that no more than "few" children will develop a lifelong interest in literature.

Answer choice (A) can be understood as "at least/at a minimum these people will [develop a lifelong interest in literature]," you know, everyone who spends their free time reading stories. Answer choice (A) leaves open the possibility that others will too!

We don't want that. We want to say, "That's it. These 'few children' are the only ones who will develop a lifelong interest in literature."

Formally:

(A) free time reading stories :arrow: lifelong interest in literature
(B) lifelong interest in literature :arrow: free time reading stories

They are mistaken reversals of each other. We need "free time reading stories" to be the necessary condition.

I hope this helps!
User avatar
 scarlett1233
  • Posts: 3
  • Joined: Oct 07, 2023
|
#103653
Hi, I often see different staff members here providing explanations that say in order to justify a question, we need certain parts of the correct answers to be necessary conditions and other parts to be sufficient for reaching a conclusion. I'm really confused about this. Could someone please explain this rule in more detail and provide guidance on how to apply it when looking for correct answers? Alternatively, could you direct me to an article or class that fully explains this rule? Thanks a lot.
User avatar
 Jeff Wren
PowerScore Staff
  • PowerScore Staff
  • Posts: 947
  • Joined: Oct 19, 2022
|
#103720
Hi Scarlett,

This is a concept that we call The Justify Formula. It is discussed in lesson 4 of The PowerScore LSAT Course and in chapter 10 of "The Logical Reasoning Bible."

You can also find information about it here:

https://blog.powerscore.com/lsat/unders ... questions/

And in episode 12 of the podcast, which can be found here:

https://blog.powerscore.com/lsat/lsat-p ... questions/

The basic idea is that the correct answer to a Justify question, when added to the premises already given in the argument, will 100% get you to/prove the conclusion.

Since arguments in Justify questions often involve conditional reasoning, the correct answer often involves a conditional statement with a term from a premise in the sufficient condition and a term from the conclusion in the necessary condition.

Here's a very simple example to illustrate how it works. Imagine we have the following argument.

Premise: John has a Snickers.
Conclusion: Therefore, John has a candy bar.

What answer would prove this conclusion 100%?

The answer is:

If someone has a Snickers, then that person has a candy bar.

If you add this statement into the argument, it 100% gets you from the premise to the conclusion. In other words, it Justifies the argument. Of course, the correct answer could be stated in the form of the contrapositive of what we are looking for (as it is in question 22), so be on the lookout for this as well.

If the answer had been stated backwards, what we call a Mistaken Reversal, it would not Justify the conclusion because it would not get us from the premise to the conclusion.

The following answer would be incorrect:

If someone has a candy bar, then that person has a Snickers.

Of course, arguments can get much more complex than the example given, but the idea is the same. Properly diagramming the conditional statements (when there is conditional reasoning) will help spot the missing "link" in the logical chain.
User avatar
 scarlett1233
  • Posts: 3
  • Joined: Oct 07, 2023
|
#103768
Thank you, I understand it now. :-D

Get the most out of your LSAT Prep Plus subscription.

Analyze and track your performance with our Testing and Analytics Package.